Laura Damone
Thanks Received: 94
Atticus Finch
Atticus Finch
 
Posts: 468
Joined: February 17th, 2011
 
 
 

Re: Q10 - Ms. Sandstrom's newspaper column

by Laura Damone Fri Dec 31, 1999 8:00 pm

What does the Question Stem tell us?
"properly inferred if which one of the following" = Sufficient Assumption

Break down the Stimulus:
Ms. S wrote a newspaper column that resulted in property damage on the Mendels' farm. From this, the argument concludes that Ms. S should pay for this damage if she could have reasonably expected her column to have resulted in said damage. Note that this conclusion is conditional. It doesn't conclude "she should pay." It concludes that if a certain condition is met, she should pay.

Any prephrase?
The first step in prephrasing a Sufficient Assumption question is always to look for any new concepts in the conclusion. Since they're not drawn from a premise, they have to feature in the correct answer if we're going to make the argument valid, which is our task. Here, we have two new concepts: the prescriptive claim that someone should pay, and the condition under which that is said to be so--the reasonable expectation of the column to lead to damage. Our correct answer will have to include both concepts, and in all likelihood will be a principle that unites them.

Correct answer:
A

Answer choice analysis:
A) Correct! A match for our prephrase. It broadens the scope from Ms. S to all people, but since Ms. S is a person, it applies to Ms. S and allows us to conclude that if she could have reasonably expected her article to lead to a stampede at the Mendel's farm, she should pony up and pay for the damages.

B) Tempting! (B) is the reverse of what we need. Note the “only if” where (A) gave us “if.” Also, note the subtle shift between the stimulus, where the condition is “she could have reasonably expected…” and (B), where the condition is “one expected.”

C) doesn't mention either of our new concepts.

D) Tempting! If the last line of the stimulus were not the conclusion, but was instead a premise offered in support of the conclusion "she should pay," this would be the correct answer. But in fact, our conclusion is conditional, so we don't need to know whether or not she fulfills the condition to draw it.

E) Also pretty tempting. But again, we don't need Ms. S to fulfill the condition of having a reasonable expectation of the disastrous results of her actions because the conclusion is simply that if she could, she should be held accountable. What's more, the Mendels beliefs about Ms. S are totally irrelevant, and can serve as a red flag alerting us to the incorrectness of this answer choice.

Takeaway/Pattern: Be careful not to confuse a conditional prescription with an absolute one, and if a Sufficient Assumption conclusion has a new concept, it has to be part of the right answer. If it has two new concepts, they both have to be part of the right answer.

#officialexplanation
Laura Damone
LSAT Content & Curriculum Lead | Manhattan Prep
 
irini101
Thanks Received: 1
Forum Guests
 
Posts: 49
Joined: August 30th, 2011
 
 
 

Q10 - Ms. Sandstrom's newspaper column

by irini101 Thu Nov 03, 2011 7:37 pm

Could any one please point out what's the flaw in D exactly? Is it a scope shift from D "knew...could incite trespassing...could result in damage" to argument "reasonably expect would lead to damage"?

Thanks in advance!
User avatar
 
bbirdwell
Thanks Received: 864
Atticus Finch
Atticus Finch
 
Posts: 803
Joined: April 16th, 2009
 
This post thanked 1 time.
 
 

Re: Q10 - Ms. Sandstrom's newspaper column

by bbirdwell Wed Nov 09, 2011 10:53 pm

Focus on making the conclusion valid. Our task is to take the given facts, and use a sufficient assumption (the answer choices) to guarantee the conclusion's validity.

The conclusion here is this:
She should pay.

It's based on the condition:
If she knew it would cause damage.

So the argument's structure is this:
knew it would cause damage --> should pay

Our job is to connect the two sides together, guaranteeing that "knowing it would cause damage" leads to "should pay."

That's what (A) does, exactly.

(B) is wrong because it says "only if" and that's not a good match for what we want our argument to do. In fact it's backwards.

(C) This is not sufficient to guarantee the conclusion. (notice the lack of any mention of Sandstrom having to pay)

(D) This merely repeats part of the argument.

(E) This is not sufficient to guarantee the conclusion (notice the lack of any mention of Sandstrom having to pay)
I host free online workshop/Q&A sessions called Zen and the Art of LSAT. You can find upcoming dates here: http://www.manhattanlsat.com/zen-and-the-art.cfm
 
nonenone
Thanks Received: 0
Forum Guests
 
Posts: 2
Joined: March 07th, 2012
 
 
 

Re: Q10 - Ms. Sandstrom's newspaper column

by nonenone Wed Mar 07, 2012 10:41 pm

I'm still confused on D:

The argument says:
Conclusion: Ms. S should pay

Reason for conclusion: IF she could have reasonably expected her column would lead people to damage the Mendel's farm.

In other words the argument says:
(evidence)IF reasonably expected the column would lead to damage ---> (conclusion)Ms. S should pay


I picked D because I interpreted it as satisfying the sufficient condition by showing she should have reasonably expected her column could cause damage. Therefore she had to pay.

I liked A but thought A just repeated the argument and didn't satisfy the sufficient condition the way D did. I thought A just repeated that the conclusion would follow IF the sufficient condition was met but that it didn't actually satisfy the sufficient condition the way D did.

This was the only question I got wrong in this section and for a #10, it is bugging me. Please help clarify further. Thanks in advance.
User avatar
 
ManhattanPrepLSAT1
Thanks Received: 1909
Atticus Finch
Atticus Finch
 
Posts: 2851
Joined: October 07th, 2009
 
This post thanked 4 times.
 
 

Re: Q10 - Ms. Sandstrom's newspaper column

by ManhattanPrepLSAT1 Mon Mar 12, 2012 4:58 pm

nonenone Wrote:In other words the argument says:
(evidence)IF reasonably expected the column would lead to damage ---> (conclusion)Ms. S should pay


I picked D because I interpreted it as satisfying the sufficient condition by showing she should have reasonably expected her column could cause damage. Therefore she had to pay.

Great point nonenone! But remember the conclusion contains the issue of paying for the damage, not the evidence. And this is exactly Brian's point! Answer choice (D) simply repeats the trigger (Ms. Sampson could have reasonably expected her column to lead others to cause damage), but doesn't guarantee the outcome (that she should pay).

The assumption should bridge the gap between them. And here's where I think you've misinterpreted the argument. I think your point is that there's a relationship:

If it could be reasonably expected that her column would lead to damage, then Ms. Sampson should pay.

And a conclusion:

She should pay.

And that we're looking for an assumption that will trigger the relationship, so that we can conclude that Ms. Sampson should pay.

But your interpretation would suggest the evidence states something about paying for the damage. But the evidence never mentions payment of any sort. Instead, it is the relationship between a reasonable expectation of leading to damage and payment for that damage that is the conclusion we are trying to justify.

To put this in another way, the conclusion does not say that Ms. Sampson should pay. The conclusion actually says that if Ms. Sampson could have reasonably expected her column to lead to damage that she should pay.

Evidence: Ms. Sampson's column led to others damaging property.
Conclusion: If Ms. Sampson could have reasonably expected her column to lead to damage, then she should pay.

Thus, we don't simply want to hit the trigger in the relationship of the conclusion, so as to justify that she should actually pay. Instead, we want an assumption that justifies the relationship between her reasonably expecting her column to lead to damage and her paying for that damage.

Answer choice (D) does not justify the relationship, since it does not say anything about her needing to pay for the damage. Pairing answer choice (D) with the evidence that Ms. Sampson's column led to the property's damage (first sentence) alone will not justify a conclusion about her paying for that damage, since nowhere amongst answer choice (D) and the first sentence is paying for something discussed.

Hope that helps!
User avatar
 
bbirdwell
Thanks Received: 864
Atticus Finch
Atticus Finch
 
Posts: 803
Joined: April 16th, 2009
 
This post thanked 1 time.
 
 

Re: Q10 - Ms. Sandstrom's newspaper column

by bbirdwell Mon Mar 12, 2012 5:02 pm

Thanks Matt! I'll add a bit, too. I actually over-simplified things a bit above.

Technically, the conclusion includes the condition. So our conclusion would be:
if she knew there would be damage --> she should pay.

We need to support this ENTIRE conditional statement in order to answer the question.

(D) is tempting, but all it does is essentially flip the conditional trigger, leading us to conclude "she should pay" -- it does not support the conditional conclusion, though. We need to support the whole idea that "IF she knew... she should pay."

(A) may sound kind of obvious by sort of paralleling the logic in it's entirety, but that's exactly what we've got to do in a situation like this. It's almost like adding a principle. The argument says X-->Y (specific situation), and to make it valid we say X-->Y (general situation).
I host free online workshop/Q&A sessions called Zen and the Art of LSAT. You can find upcoming dates here: http://www.manhattanlsat.com/zen-and-the-art.cfm
 
ptewarie
Thanks Received: 36
Forum Guests
 
Posts: 38
Joined: October 01st, 2012
 
 
 

Re: Q10 - Ms. Sandstrom's newspaper column

by ptewarie Sat Aug 31, 2013 11:00 pm

One thing to keep in mind(and a reason why many people are getting this wrong is ) is that the conclusion is not just
" Ms. Sandstorm should pay"
It is
"Ms. Sandstorm should pay IF she could have reasonably expected that the column would lead to damage"

There's a difference for the author is NOT saying she should pay.

In sufficient assumption questions, the right answer choice, added to the premise, will logically lead to the conclusion.

so :

Premise:
Ms. S's article led many people to damage the property

Conclusion:
Ms. Sandstorm should pay IF she could have reasonably expected that the column would lead to damage


The only AC that would logically bridge Premise to conclusion is that " if reasonably expected- pay" which is A.
 
rickytucker
Thanks Received: 3
Forum Guests
 
Posts: 13
Joined: August 26th, 2013
 
 
 

Re: Q10 - Ms. Sandstrom's newspaper column

by rickytucker Fri Sep 20, 2013 12:59 am

Since sufficient assumption questions are asking for the opposite of necessary assumption questions (correct AC --> valid conclusion), would negating the conclusion be a worthwhile technique during the test?

In this case, negating the conclusion:
if knew --> ~pay

kills AC (A) but leaves (D) unscathed:
(A) if knew --> pay
(D) knew could incite damage
 
gaheexlee
Thanks Received: 10
Elle Woods
Elle Woods
 
Posts: 55
Joined: May 27th, 2014
 
This post thanked 1 time.
 
 

Re: Q10 - Ms. Sandstrom's newspaper column

by gaheexlee Mon Sep 15, 2014 3:15 pm

For anyone who was also confused as to how to pick between (A) and (D), take a look at PT 36, S1, #18.

It has a stimulus structure almost identical to this one, as well as similar answer choices. Reading Matt and Brian's explanations reallyyyy allowed me to see the subtle difference between the two.
 
joshgra92
Thanks Received: 0
Vinny Gambini
Vinny Gambini
 
Posts: 2
Joined: April 06th, 2016
 
 
 

Re: Q10 - Ms. Sandstrom's newspaper column

by joshgra92 Tue May 17, 2016 9:18 pm

I was wondering if someone could talk more about what makes B.) wrong. It is almost identical to the correct answer A.)

Thanks in advanced.
 
Fazzvm56
Thanks Received: 0
Vinny Gambini
Vinny Gambini
 
Posts: 5
Joined: January 27th, 2016
 
 
 

Re: Q10 - Ms. Sandstrom's newspaper column

by Fazzvm56 Sat May 28, 2016 2:21 pm

I also wanted a little clarification on what makes B wrong. I chose A, but I'm still not sure I'm sold on what makes B wrong exactly? Is it because B forces Ms. Sandstrom to actually have known that her action would cause the damage? Whereas A only makes it reasonably expected, if she had thought about it?
User avatar
 
ohthatpatrick
Thanks Received: 3808
Atticus Finch
Atticus Finch
 
Posts: 4661
Joined: April 01st, 2011
 
This post thanked 1 time.
 
 

Re: Q10 - Ms. Sandstrom's newspaper column

by ohthatpatrick Thu Jun 02, 2016 2:45 pm

The simplest way to tell (A) from (B) comes down to the difference between "if" and "only if".

"If", "only if", and "unless" are the three most common conditional triggers on the test, so you better know how they each work.

"If" = left side idea
"only if" = right side idea
"unless" = 'if not'

If X then Y
X --> Y

Only if X then Y
Y --> X

Unless X, Y
~X --> Y

So (A) and (B) are reversed versions of each other (notwithstanding the other nuance of "reasonable expectation" vs. "expectation")

(A)
if "reasonably expect action would lead others to damage" then "should pay for that damage"

(B)
if "should pay for that damage" then "expected that action would lead others to damage"

The #1 trick of Sufficient Assumption answer choices is giving you two different versions of the ideas that need to be linked.

We always need to see
PREM --> CONC

and trap answers like to tempt us with
CONC --> PREM

It will always, always, always be wrong to see
"IF conclusion .... "

(Note: it's okay to see the contrapositive of PREM --> CONC, which would look like "IF not-conclusion, THEN not-premise")

Hope this helps.
 
erikwoodward10
Thanks Received: 9
Elle Woods
Elle Woods
 
Posts: 69
Joined: January 26th, 2014
 
 
 

Re: Q10 - Ms. Sandstrom's newspaper column

by erikwoodward10 Wed Jul 06, 2016 3:42 pm

Would answer choice E be a necessary assumption?
 
VendelaG465
Thanks Received: 0
Elle Woods
Elle Woods
 
Posts: 66
Joined: August 22nd, 2017
 
 
 

Re: Q10 - Ms. Sandstrom's newspaper column

by VendelaG465 Tue Oct 24, 2017 9:38 pm

Hi,

I'm still confused as to why B couldn't work. I wrote if she could've expected that the column would lead people to damage the Mendel's farm -----> then Ms. S should pay.

Wouldn't A be an illegal negation?
I was under the impression that we're never able to infer A--->B but only -B ---> -A.
& 2nd I read your post about how certain answer choices start with "If conclusion..." is wrong. but doesn't choice A start with the conclusion?
User avatar
 
ohthatpatrick
Thanks Received: 3808
Atticus Finch
Atticus Finch
 
Posts: 4661
Joined: April 01st, 2011
 
 
 

Re: Q10 - Ms. Sandstrom's newspaper column

by ohthatpatrick Wed Oct 25, 2017 1:22 pm

* If Y happens, X happens
* X happens if Y happens
both are symbolized: Y --> X

* Only if Y happens does X happen
* X happens only if Y happens
both are symbolized: X --> Y

Your confusion seems to be coming from not seeing that (A) is using "if" and (B) is using "only if".

"if" = attached to left side ideas
"only if" = attached to right side ideas

Your version of (B) is incorrect because (B) says
"only if one expected the action would lead to damage"

Thus, "expected the action would lead to damage" must be the right side idea. We put the other idea on the left and get
"Should pay ---> expected action would lead to damage"

In (A), it says
"if one could have reasonably expected damage"

Thus, "expected damage" is a left side idea. We put the other idea on the right and get:
"Reasonably expected damage ---> should pay"
 
CharlesS800
Thanks Received: 2
Vinny Gambini
Vinny Gambini
 
Posts: 17
Joined: July 09th, 2017
 
 
 

Re: Q10 - Ms. Sandstrom's newspaper column

by CharlesS800 Sun Jul 08, 2018 12:56 pm

I struggled with this question during a recent practice test. I incorrectly selected D as the answer. Upon review, I selected A instead. I think part of what threw me off about this question was the portion of the second sentence preceding the premise, "if, as the Mendels claim." While taking the PT, I assumed that this was part of the premise and created a conditional statement. However, upon review, I reversed my thinking. I decided that instead of the premise beginning with an if, it was simply:

[Ms. Sandstrom] could have reasonably expected the column would lead to damage on the Mendel's farm.

The conclusion I identified was: Ms. Sandstrom should pay for the damage.

While originally taking this PT, I had kept A as a possible answer choice and I did the same upon reviewing the question. I discarded B because it was the reverse of what I wanted, the "only if" portion of the statement meant that this question was a reversal of the answer that I was looking for. I got rid of answer C because it was out of scope. I kept answer D upon review as well. I discarded answer E because it didn't address the crux of what I saw as the gap, the transition from expecting damage as a result of the column to PAYING for the damage. Because it didn't' address the paying portion, I got rid of it. I then examined D again. I put it in the middle of my premise and conclusion and although it sounded alright, it didn't fully bridge the gap between the premise and conclusion like answer choice A did. I think one other thing that hung me up about this question was the simplicity of answer A - it fully bridged the argument with clear language, something that I think I did not expect, as I thought the correct answer would be a little trickier. I think in the future with questions like this, it is important to make sure I fully understand the full argument before jumping into answer choices and remembering the sufficient assumptions will fully bridge the gap in the argument.